[Complex Analysis] Finding a conformal map

Click For Summary
The discussion focuses on finding a conformal map from the vertical strip defined by -1 < Re(z) < 1 to the unit disk. It emphasizes that conformal mappings can be achieved using linear fractional or Möbius transformations, which are defined by the images of three points. The user expresses frustration in finding a suitable transformation but recalls that transformations from other geometric shapes, like half-planes or disks, to the unit disk are known. A suggestion is made to consider the exponential function, which maps a strip to the upper half-plane, as a potential approach. The conversation highlights the challenge of the problem while exploring related concepts in complex analysis.
nonequilibrium
Messages
1,412
Reaction score
2

Homework Statement


I have to find a conformal map from \Omega = \{ z \in \mathbb C | -1 &lt; \textrm{Re}(z) &lt; 1 \}
to the unit disk D(0,1)

Homework Equations


an analytical function f is conformal in each point where the derivative is non-vanishing
specifically, we can think of linear fractionals/mobius transformations, which are conformal everywhere and determined by the image of three numbers

The Attempt at a Solution


I've busted my brain on this one, but I can't think of anything that will transform these two vertical boundary lines into something useful. Note that I do not have to transform it to the unit disk; also a half/quarter-plane, a disk with a slit cut out, half a disk, will do, as I already know (from previous exercises) conformal transformations from those sets to the unit disk
 
Physics news on Phys.org
If you take the set {z : 0<Im(z)<pi} then exp(z) maps that strip into the upper half plane, right? That's a lot like your problem.
 
Oh jeesh I can't believe I didn't see that x_x

Let's hope I get all the stupidity out before the exam

Thank you!
 
Question: A clock's minute hand has length 4 and its hour hand has length 3. What is the distance between the tips at the moment when it is increasing most rapidly?(Putnam Exam Question) Answer: Making assumption that both the hands moves at constant angular velocities, the answer is ## \sqrt{7} .## But don't you think this assumption is somewhat doubtful and wrong?

Similar threads

Replies
2
Views
1K
  • · Replies 4 ·
Replies
4
Views
2K
  • · Replies 1 ·
Replies
1
Views
2K
  • · Replies 5 ·
Replies
5
Views
2K
  • · Replies 2 ·
Replies
2
Views
3K
  • · Replies 2 ·
Replies
2
Views
2K
  • · Replies 1 ·
Replies
1
Views
2K
Replies
8
Views
3K
  • · Replies 3 ·
Replies
3
Views
2K
  • · Replies 25 ·
Replies
25
Views
2K